[Rozgrzewka OM][MIX][Nierówności] Nierówności

Zadania z kółek matematycznych lub obozów przygotowujących do OM. Problemy z minionych olimpiad i konkursów matematycznych.
Regulamin forum
Wszystkie tematy znajdujące się w tym dziale powinny być tagowane tj. posiadać przedrostek postaci [Nierówności], [Planimetria], itp.. Temat może posiadać wiele różnych tagów. Nazwa tematu nie może składać się z samych tagów.
Zahion
Moderator
Moderator
Posty: 2095
Rejestracja: 9 gru 2012, o 19:46
Płeć: Mężczyzna
Lokalizacja: Warszawa, mazowieckie
Podziękował: 139 razy
Pomógł: 504 razy

Re: [Rozgrzewka OM][MIX][Nierówności] Nierówności

Post autor: Zahion »

W nieujemnych \(\displaystyle{ a \ge b \ge c}\) takich, że \(\displaystyle{ a + b + c = 1}\) wykazać:
\(\displaystyle{ a^{4}\left( b - c \right) + b^{4}\left( c - a\right) + c^{4}\left( a - b \right) \ge a\left( b - c\right)^{3} + b\left( c - a \right)^{3} + c\left( a - b\right)^{3}}\)
bosa_Nike
Użytkownik
Użytkownik
Posty: 1665
Rejestracja: 16 cze 2006, o 15:40
Płeć: Kobieta
Podziękował: 71 razy
Pomógł: 445 razy

[Rozgrzewka OM][MIX][Nierówności] Nierówności

Post autor: bosa_Nike »

Już po OMJ, a skoro za poprzednie się nikt z uczestników tu nie wziął, to winy za niewielką popularność łańcuszka nie ma co zrzucać na dużą trudność zadań.
Ukryta treść:    
Dla \(\displaystyle{ a,b,c>0}\) udowodnij \(\displaystyle{ \frac{a^2}{b^2}+\frac{b^2}{c^2}+\frac{c^2}{a^2}+\frac{2abc}{a^3+b^3+c^3}\ge\frac{11}{3}}\)
Awatar użytkownika
Premislav
Użytkownik
Użytkownik
Posty: 15687
Rejestracja: 17 sie 2012, o 13:12
Płeć: Mężczyzna
Lokalizacja: Warszawa
Podziękował: 196 razy
Pomógł: 5220 razy

Re: [Rozgrzewka OM][MIX][Nierówności] Nierówności

Post autor: Premislav »

Ukryta treść:    
Nowe zadanie:
niech liczby \(\displaystyle{ a_0, \ a_1, \ldots a_n}\) z przedziału \(\displaystyle{ \left( 0, \frac \pi 2\right)}\) spełniają warunek \(\displaystyle{ \sum_{i=0}^{n}\tg\left( a_i-\frac \pi 4\right) \ge n-1}\).
Proszę wykazać, że \(\displaystyle{ \prod_{i=0}^{n} \tg a_i\ge n^{n+1}}\).
bosa_Nike
Użytkownik
Użytkownik
Posty: 1665
Rejestracja: 16 cze 2006, o 15:40
Płeć: Kobieta
Podziękował: 71 razy
Pomógł: 445 razy

[Rozgrzewka OM][MIX][Nierówności] Nierówności

Post autor: bosa_Nike »

Do poprzedniego:    
Bieżące:    
Awatar użytkownika
Premislav
Użytkownik
Użytkownik
Posty: 15687
Rejestracja: 17 sie 2012, o 13:12
Płeć: Mężczyzna
Lokalizacja: Warszawa
Podziękował: 196 razy
Pomógł: 5220 razy

Re: [Rozgrzewka OM][MIX][Nierówności] Nierówności

Post autor: Premislav »

Świetnie. Jakby ktoś był zainteresowany, jest to zadanie 3. z USAMO 1998, rozwiązanie wzorcowe (zapewne) można zobaczyć

Kod: Zaznacz cały

https://artofproblemsolving.com/wiki/index.php/1998_USAMO_Problems/Problem_3
.
Zadajesz (chyba że nie chcesz), bosa_Nike.
Awatar użytkownika
Sylwek
Użytkownik
Użytkownik
Posty: 2716
Rejestracja: 21 maja 2007, o 14:24
Płeć: Mężczyzna
Lokalizacja: Warszawa
Podziękował: 160 razy
Pomógł: 657 razy

Re: [Rozgrzewka OM][MIX][Nierówności] Nierówności

Post autor: Sylwek »

Pozwolę sobie wkleić podobne rozwiązanie tego z tangensem, bo zacząłem pisać wcześniej i miałem to dziś umieścić
To z tangensem:    
I nawet miałem przygotowane następne, więc się wetnę:

Pokaż, że dla dowolnych \(\displaystyle{ a, b, c > 0}\) zachodzi nierówność
\(\displaystyle{ \sqrt{a^2-ab+b^2} + \sqrt{b^2-bc+c^2} \ge \sqrt{a^2+ac+c^2}}\).
bosa_Nike
Użytkownik
Użytkownik
Posty: 1665
Rejestracja: 16 cze 2006, o 15:40
Płeć: Kobieta
Podziękował: 71 razy
Pomógł: 445 razy

[Rozgrzewka OM][MIX][Nierówności] Nierówności

Post autor: bosa_Nike »

No nie, to jest nieludzkie. Nie dość, że się pod koniec redagowania odpowiedzi zorientowałam, że rozwiązałam inne zadanie, musiałam przerobić całe rozwiązanie i wpisać je na nowo, to jeszcze Sylwek się przyznaje, że miał, ale nie wrzucił. Proszę mi się na przyszłość pospieszyć, bo ja mam okna do mycia.
Bieżące:    
Dla \(\displaystyle{ a,b,c>0}\), takich że \(\displaystyle{ a+b+c=3}\) udowodnij

\(\displaystyle{ \frac{1}{4}\left(\frac{abc}{3}-1\right)(abc-1)\ge (1-a)(1-b)(1-c)}\)
Awatar użytkownika
Premislav
Użytkownik
Użytkownik
Posty: 15687
Rejestracja: 17 sie 2012, o 13:12
Płeć: Mężczyzna
Lokalizacja: Warszawa
Podziękował: 196 razy
Pomógł: 5220 razy

Re: [Rozgrzewka OM][MIX][Nierówności] Nierówności

Post autor: Premislav »

Ukryta treść:    
Prosiłbym, żeby ktoś to sprawdził, bo rzadko coś mi wychodzi tak low effort.
bosa_Nike
Użytkownik
Użytkownik
Posty: 1665
Rejestracja: 16 cze 2006, o 15:40
Płeć: Kobieta
Podziękował: 71 razy
Pomógł: 445 razy

[Rozgrzewka OM][MIX][Nierówności] Nierówności

Post autor: bosa_Nike »

Wg mnie rozwiązanie jest prawidłowe i bardzo fajne.
Awatar użytkownika
Premislav
Użytkownik
Użytkownik
Posty: 15687
Rejestracja: 17 sie 2012, o 13:12
Płeć: Mężczyzna
Lokalizacja: Warszawa
Podziękował: 196 razy
Pomógł: 5220 razy

Re: [Rozgrzewka OM][MIX][Nierówności] Nierówności

Post autor: Premislav »

Dzięki za sprawdzenie. Wrzucam nowe zadanie:
niech \(\displaystyle{ n\in \NN^+}\) będzie ustalone. Proszę znaleźć wszystkie takie \(\displaystyle{ m\in \NN^+}\), że nierówność \(\displaystyle{ \frac{1}{a^n}+\frac{1}{b^n}\ge a^m+b^m}\) zachodzi dla wszystkich dodatnich liczb \(\displaystyle{ a,b}\) spełniających warunek \(\displaystyle{ a+b=2.}\)
PokEmil
Użytkownik
Użytkownik
Posty: 164
Rejestracja: 25 mar 2017, o 15:35
Płeć: Mężczyzna
Lokalizacja: Zamość
Podziękował: 19 razy
Pomógł: 20 razy

Re: [Rozgrzewka OM][MIX][Nierówności] Nierówności

Post autor: PokEmil »

Co do zadania Sylwka:
Ukryta treść:    
Awatar użytkownika
Sylwek
Użytkownik
Użytkownik
Posty: 2716
Rejestracja: 21 maja 2007, o 14:24
Płeć: Mężczyzna
Lokalizacja: Warszawa
Podziękował: 160 razy
Pomógł: 657 razy

Re: [Rozgrzewka OM][MIX][Nierówności] Nierówności

Post autor: Sylwek »

Oboje bardzo pięknie!

Zachęcam powymyślać podobne nierówności i ciachać kąt \(\displaystyle{ 60^o, 75^o, 90^o, 105^o, 120^o, 135^o, 150^o, 165^o}\) czy \(\displaystyle{ 180^o}\) na części mające \(\displaystyle{ 30^o, 45^o, 60^o, 90^o, 120^o, 135^o}\) czy \(\displaystyle{ 150^o}\).

Lub inne, mniej przyjemne dla oka wartości.
bosa_Nike
Użytkownik
Użytkownik
Posty: 1665
Rejestracja: 16 cze 2006, o 15:40
Płeć: Kobieta
Podziękował: 71 razy
Pomógł: 445 razy

[Rozgrzewka OM][MIX][Nierówności] Nierówności

Post autor: bosa_Nike »

Finał OM przeszedł bez echa, tutaj też wiele się już nie zdarzy - może warto pomyśleć o zmianie zadania.

Nierówność zamieszczona przez Premislava to zadanie 2. z drugiego dnia zawodów OM z Bułgarii z 2008 roku - do znalezienia w dziale Contests na aops. Ja porzuciłam ten problem po zrobieniu ok. połowy z trzypunktowego planu alternatywnego rozwiązania, odstręczona rozmiarem i bez przekonania o skuteczności pomysłu.
Awatar użytkownika
Premislav
Użytkownik
Użytkownik
Posty: 15687
Rejestracja: 17 sie 2012, o 13:12
Płeć: Mężczyzna
Lokalizacja: Warszawa
Podziękował: 196 razy
Pomógł: 5220 razy

Re: [Rozgrzewka OM][MIX][Nierówności] Nierówności

Post autor: Premislav »

Mnie się nie chce pisać rozwiązania, wiec po prostu zmienię nierówność.

Nowa (jak już ten wątek poruszyłaś; no cóż, leniwy jestem):
niech liczby dodatnie \(\displaystyle{ a_0, a_1\ldots a_n}\) będą takie, że \(\displaystyle{ a_0\in \ZZ}\) oraz \(\displaystyle{ a_i\le a_{i-1}+1}\) dla \(\displaystyle{ i\in\left\{ 1,2\ldots n\right\}}\). Proszę znaleźć jak najmniejszą stałą dodatnią \(\displaystyle{ C}\), dla której nierówność
\(\displaystyle{ n\le Ca_0 \sum_{i=1}^{n} \frac{1}{a_i}}\)
zachodzi dla dowolnych \(\displaystyle{ a_0, \ldots a_n}\) spełniających powyższe warunki.-- 20 kwi 2019, o 14:43 --Jezuu, nie patrzyłem przez tydzień na ten wątek i teraz widzę, że zgubiłem istotne założenie, że
\(\displaystyle{ \sum_{i=1}^{n}\frac{1}{a_i}\le 1}\). Dopiszę jutro, bo teraz pies z kulawą nogą tego nie zauważy.
Awatar użytkownika
Premislav
Użytkownik
Użytkownik
Posty: 15687
Rejestracja: 17 sie 2012, o 13:12
Płeć: Mężczyzna
Lokalizacja: Warszawa
Podziękował: 196 razy
Pomógł: 5220 razy

Re: [Rozgrzewka OM][MIX][Nierówności] Nierówności

Post autor: Premislav »

Bardzo przepraszam, nieuważnie przepisywałem treść, zgubiłem założenie o \(\displaystyle{ \sum_{i=1}^{n}\frac{1}{a_i}\le 1}\) (bez tego oczywiście nie działa!). Naprawdę mi wstyd. Ale i tak myślę, że wrzucanie tego zadania tutaj nie miało większego sensu (bo trochę za łatwe, a jeszcze zepsułem treść).
... m70_3r.pdf
Zadanie piąte (bardzo łatwo poprawić stałą na \(\displaystyle{ 3}\), a znając trochę analizy na \(\displaystyle{ e}\), natomiast to, co w tym było dla mnie najzabawniejsze, to że gubiąc pewien warunek, myślałem po chwili, że poprawiłem stałą na \(\displaystyle{ 2}\), a otóż nie, gdyż aż tak dobrze się nie da). Swoją drogą trochę słabo, że nie dość, że idzie na samej AM-HM bez żadnego pomysłu, to jeszcze forma nierówności narzuca taką drogę.

Może teraz coś takiego na rozruszanie:
niech \(\displaystyle{ n>2, \ x_1, x_2, \ldots x_n\ge 0}\) oraz \(\displaystyle{ x_{i}+x_{i+1}>0}\) dla \(\displaystyle{ i=1,2\ldots n}\), przy czym przyjmujemy, że \(\displaystyle{ x_{n+1}=x_1}\). Proszę udowodnić, że
\(\displaystyle{ 1< \sum_{i=1}^{n}\frac{x_i}{x_{i}+x_{i+1}}<n-1}\)
ODPOWIEDZ